Sie sind auf Seite 1von 21

CHAPTER 45 COMPLEX NUMBERS

EXERCISE 187 Page 510

1. Solve the quadratic equation: x2 + 25 = 0

Since x 2 + 25 =
0 then x 2 = −25

i.e. x = −25 = (−1)(25) = −1 25 = j 25

from which, x = ± j5

2. Solve the quadratic equation: x2 – 2x + 2 = 0

Since x2 – 2x + 2 = 0 then

−−2± [(−2)2 − 4(1)(2)] 2 ± −4 2 ± (−1)(4) 2 ± (−1) (4) 2 ± j (4)


=x = = = =
2(1) 2 2 2 2

2 4 2 2
= ±j =± j = 2 ± j1 = 1 ± j
2 2 2 2

3. Solve the quadratic equation: x2 – 4x + 5 = 0

Since x2 – 4x + 5 = 0 then

−−4± [(−4)2 − 4(1)(5)] 4 ± −4 4 ± (−1)(4) 4 ± (−1) (4) 4 ± j (4)


=x = = = =
2(1) 2 2 2 2

4 4 4 2
= ±j =± j = 2 ± j1 = 2 ± j
2 2 2 2

4. Solve the quadratic equation: x2 – 6x + 10 = 0

Since x2 – 6x + 10 = 0 then

−−6± [(−6)2 − 4(1)(10)] 6 ± −4 6 ± (−1)(4) 6 ± (−1) (4) 6 ± j (4)


=x = = = =
2(1) 2 2 2 2

757 © 2014, John Bird


6 4 6 2
= ±j =± j = 3 ± j
2 2 2 2

5. Solve the quadratic equation: 2x2 – 2x + 1 = 0

Since 2x2 – 2x + 1 = 0 then

−−2± [(−2)2 − 4(2)(1)] 2 ± −4 2 ± (−1)(4) 2 ± (−1) (4) 2 ± j (4)


=x = = = =
2(2) 4 4 4 4

2 4 2 2
= ±j =± j = 0.5 ± j0.5
4 4 4 4

6. Solve the quadratic equation: x2 – 4x + 8 = 0

Since x2 – 4x + 8 = 0 then

−−4± [(−4)2 − 4(1)(8)] 4 ± −16 4 ± (−1)(16) 4 ± (−1) (16) 4 ± j (16)


=x = = = =
2(1) 2 2 2 2

4 16 4 4
= ±j = ± j = 2 ± j2
2 2 2 2

7. Solve the quadratic equation: 25x2 – 10x + 2 = 0

Since 25x2 – 10x + 2 = 0 then

− − 10 ± [(−10)2 − 4(25)(2)] 10 ± −100 10 ± (−1)(100) 10 ± (−1) (100)


=x = = =
2(25) 50 50 50

10 ± j (100) 10 100 10 10
= = ±j =± j = 0.2 ± j0.2
50 50 50 50 50

8. Solve the quadratic equation: 2x2 + 3x + 4 = 0

Since 2 x 2 + 3 x + 4 =0 then

−3 ± [32 − 4(2)(4)] −3 ± −23 −3 ± (−1)(23) −3 ± (−1) (23) −3 ± j (23)


=x = = = =
2(2) 4 4 4 4

758 © 2014, John Bird


3 23
=– ± j or (– 0.750 ± j1.199)
4 4

9. Solve the quadratic equation: 4t2 – 5t + 7 = 0

Since 4t2 – 5t + 7 = 0 then

−−5± [(−5)2 − 4(4)(7)] 5 ± −87 5 ± (−1)(87) 5 ± (−1) (87) 5 ± j (87)


=t = = = =
2(4) 8 8 8 8

5 87
= ±j or (0.625 ± j1.166)
8 8

1 4
10. Evaluate (a) j8 (b) – (c)
j7 2 j13

(a) j 8 = ( j 2 ) = ( −1) = 1
4 4

(b) j 7 = j × j 6 = j × ( j 2 ) = j × ( −1) = − j
3 3

1 1 1 −j −j −j −j
Hence, − =
− == = = = = –j
j 7 − j j j (− j ) − j 2 −(−1) 1

(c) j13 = j × j12 = j × ( j 2 ) = j × (−1)6 = j


6

4 2 2(− j ) − j 2 − j 2
Hence, == = = = –j2
2 j13 j j (− j ) − j 2 1

759 © 2014, John Bird


EXERCISE 188 Page 513

1. Evaluate (a) (3 + j2) + (5 – j) and (b) (–2 + j6) – (3 – j2) and show the results on an Argand diagram.

(a) (3 + j2) + (5 – j) = (3 + 5) + j(2 – 1) = 8 + j

(b) (–2 + j6) – (3 – j2) = –2 + j6 – 3 + j2 = (–2 – 3) + j(6 + 2) = –5 + j8

(8 + j) and (–5 + j8) are shown on the Argand diagram below.

2. Write down the complex conjugates of (a) 3 + j4, (b) 2 – j.

(a) The complex conjugate of 3 + j4 is: 3 – j4

(b) The complex conjugate of 2 – j is: 2 + j

3. If z = 2 + j and w = 3 – j evaluate (a) z + w (b) w – z (c) 3z – 2w

(d) 5z + 2w (e) j(2w – 3z) (f) 2jw – jz

(a) z + w = (2 + j) + (3 – j) = 2 + j + 3 – j = 5

(b) w – z = (3 – j) – (2 + j) = 3 – j – 2 – j = 1 – j2

(c) 3z – 2w = 3(2 + j) – 2(3 – j) = 6 + j3 – 6 + j2 = j5

(d) 5z + 2w = 5(2 + j) + 2(3 – j) = 10 + 5j + 6 – j2 = 16 + j3

(e) j(2w – 3z) = j[(6 – j2) – (6 +j3)] = j[6 – j2 – 6 – j3] = j(– j5) = – j 2 5 = –(– 1)5 = 5

(f) 2jw – jz = 2j(3 – j) – j(2 + j) = j6 – 2 j 2 – j2 – j 2 = j6 – 2(– 1) – j2 – (– 1) = j6 + 2 – j2 + 1 = 3 +


j4

4. Evaluate in a + jb form, given Z 1 = 1 + j2, Z2 = 4 – j3, Z 3 = –2 + j3 and Z4 = –5 – j

(a) Z 1 + Z 2 – Z 3 (b) Z 2 – Z1 + Z 4
760 © 2014, John Bird
(a) Z1 + Z 2 − Z 3 = 1 + j2 + 4 – j3 – (–2 + j3) = 1 + j2 + 4 – j3 + 2 – j3

= (1 + 4 + 2) + j(2 – 3 – 3) = 7 – j4

(b) Z 2 − Z1 + Z 4 = (4 – j3) – (1 + j2) + (–5 – j) = 4 – j3 – 1 – j2 – 5 – j

= (4 – 1 – 5) + j(–3 – 2 – 1) = – 2 – j6

5. Evaluate in a + jb form, given Z 1 = 1 + j2, Z2 = 4 – j3, Z 3 = – 2 + j3 and Z4 = –5 – j

(a) Z 1 Z 2 (b) Z 3 Z 4

(a) Z 1 Z 2 = (1 + j2)(4 – j3) = 4 – j3 + j8 – j 2 6 = 4 – j3 + j8 + 6 = 10 + j5

(b) Z3 Z 4 = (–2 + j3)(–5 – j) = 10 + j2 – j15 – j 2 3 = 10 + j2 – j15 + 3 = 13 – j13

6. Evaluate in a + jb form, given Z 1 = 1 + j2, Z2 = 4 – j3, Z 3 = –2 + j3 and Z4 = –5 – j

(a) Z 1 Z 3 + Z 4 (b) Z 1 Z 2 Z3

(a) Z1Z 3 + Z 4 = (1 + j2)(–2 + j3) + (–5 – j) = –2 + j3 – j4 + j 2 6 – 5 – j


= –2 + j3 – j4 – 6 – 5 – j = –13 – j2

(b) Z1Z 2 Z 3 = (1 + j2)(4 – j3)(–2 + j3) = (4 – j3 + j8 – j 2 6)(–2 + j3) = ( 10 + j5)(–2 + j3)

= –20 + j30 – j10 + j 2 15 = –20 + j30 –j10 –15 = –35 + j20

7. Evaluate in a + jb form, given Z 1 = 1 + j2, Z2 = 4 – j3, Z 3 = –2 + j3 and Z4 = –5 – j

Z1 Z1 + Z 3
(a) (b)
Z2 Z2 − Z4

Z1 1 + j 2 (1 + j 2)(4 + j 3) 4 + j 3 + j8 + j 2 6 4 + j 3 + j8 − 6 −2 + j11 − 2 11
(a)= = = = = = +j
Z 2 4 − j 3 (4 − j 3)(4 + j 3) 4 +3
2 2 25 25 25 25

Z1 + Z 3 (1 + j 2) + (−2 + j 3) −1 + j 5 (−1 + j 5)(9 + j 2) −9 − j 2 + j 45 + j 210 −19 + j 43


(b) = = = = =
Z 2 − Z 4 (4 − j 3) − (−5 − j ) 9 − j2 (9 − j 2)(9 + j 2) 92 + 22 85

−19 43
= +j
85 85

761 © 2014, John Bird


8. Evaluate in a + jb form, given Z 1 = 1 + j2, Z2 = 4 – j3, Z 3 = –2 + j3 and Z4 = –5 – j

Z1Z 3 Z1
(a) (b) Z 2 + + Z3
Z1 + Z 3 Z4

Z1Z 3 (1 + j 2)(−2 + j 3) −2 + j 3 − j 4 + j 2 6 −8 − j
(a) = = =
Z1 + Z 3 (1 + j 2) + (−2 + j 3) −1 + j 5 −1 + j 5

(−8 − j )(−1 − j 5) 8 + j 40 + j + j 2 5 3 + j 41 3 41
= = = = +j
(−1 + j 5)(−1 − j 5) 1 +5
2 2 26 26 26

Z1 1+ j2 (1 + j 2)(−5 + j )
(b) Z 2 + + Z 3 = (4 – j3) + + (–2 + j3) = 4 – j3 + – 2 + j3
Z4 −5 − j 52 + 12

−5 + j − j10 + j 2 2
= 4 – j3 + – 2 + j3
26

−7 − j 9 7 9
= 4 – j3 + – 2 + j3 = 2 – −j
26 26 26

52 7 9 45 9
= − −j = −j
26 26 26 26 26

1− j 1
9. Evaluate (a) (b)
1+ j 1+ j

1 − j (1 − j )(1 − j ) 1 − j − j + j 2 − j 2
=
(a) = = = 0 – j1 = – j
1 + j (1 + j )(1 − j ) 12 + 12 2

1 (1)(1 − j ) 1− j 1− j 1 1
(b) = = = = −j
1 + j (1 + j )(1 − j ) 1 + 1
2 2 2 2 2

−25  1 + j 2 2 − j 5 
10. Show that  −  = 57 + j24
2  3 + j4 −j 

1 + j 2 (1 + j 2)(3 − j 4) 3 − j 4 + j 6 − j 2 8 11 + j 2 11 2
= = = = +j
3 + j4 3 +4
2 2 25 25 25 25

2 − j 5 (2 − j 5)( j ) j2 − j2 5 5 + j2
= = = = 5 + j2
−j − j( j) − j2 1

762 © 2014, John Bird


−25  1 + j 2 2 − j 5  25  11 2   25  11   2 
L.H.S. =  − =−  + j  − (5 + j 2)  =
−  − 5  + j  − 2 
2  3 + j4 −j  2  25 25   2  25   25 

25  11 − 125   2 − 50   25  114 48 
=−  + j  =− − −j 
2  25   25   2  25 25 

25  114  25  48 
=− −  + j   = 57 + j24 = R.H.S.
2  25  2  25 

763 © 2014, John Bird


EXERCISE 189 Page 514

1. Solve: (2 + j)(3 – j2) = a + jb

(2 + j)(3 – j2) = a + jb

Hence, 6 – j4 + j3 – j 2 2 = a + jb

i.e. 8 – j1 = a + jb

Thus, a = 8 and b = –1

2+ j
2. Solve: = j(x + jy)
1− j

2+ j (2 + j )(1 + j )
= j ( x + jy ) hence, = j ( x + jy )
1− j (1 − j )(1 + j )
2 + j2 + j + j2
i.e. = jx + j 2 y
12 + 12
1 + j3
i.e. = jx − y
2
1 3
i.e. + j = –y + jx
2 2
3 1
Hence, x= and y= −
2 2

3. Solve: (2 – j3) = (a + b)

(2 − j 3) = (a + jb)

( 2 − j 3)
2
Squaring both sides gives: =
a + jb

(2 – j3)(2 – j3) = a + jb
i.e. 4 – j6 – j6 + j 2 9 = a + jb
i.e. –5 – j12 = a + jb
Hence, a = –5 and b = –12

4. Solve: (x – j2y) – (y – jx) = 2 + j

764 © 2014, John Bird


(x – j2y) – (y – jx) = 2 + j

Hence, (x – y) + j(– 2y + x) = 2 + j

i.e. x–y=2 (1)


and x – 2y = 1 (2)

(1) – (2) gives: y=1

Substituting in (1) gives: x – 1 = 2 from which, x = 3

5. If Z = R + jωL + 1/jωC, express Z in (a + jb) form when R = 10, L = 5, C = 0.04 and ω = 4

1 1 6.25 6.25(− j )
Z = R + jωL + = 10 + j(4)(5) + = 10 + j20 + = 10 + j20 +
jωC j (4)(0.04) j j (− j )

6.25
= 10 + j20 – = 10 + j20 – j6.25
− j2
= 10 + j13.75

765 © 2014, John Bird


EXERCISE 190 Page 517

1. Determine the modulus and argument of (a) 2 + j4 (b) –5 – j2 (c) j(2 – j)

(a) 2 + j4 lies in the first quadrant as shown below

Modulus, r = 42 + 22 = 4.472

4
Argument, θ = tan −1 = 63.43°
2

(b) –5 – j2 lies in the third quadrant as shown below

Modulus, r = 52 + 22 = 5.385

2
α = tan −1 = 21.80°
5

Hence, argument, θ = –(180° – 21.80°) = –158.20°

(c) j(2 – j) = j2 – j 2 = j2 + 1 or 1 + j2

1 + j2 lies in the first quadrant as shown below.

Modulus, r = 12 + 22 = 2.236

2
Argument, θ = tan −1 = 63.43°
1
766 © 2014, John Bird
2. Express in polar form, leaving answers in surd form:

(a) 2 + j3 (b) –4 (c) –6 + j

(a) 2 + j3 From the diagram below, r = 22 + 32 =13

3
and θ =tan −1   =56.31° or 56°19 '
 
2

Hence, 2 + j3 = 13∠56.31° in polar form

(b) –4 = –4 + j0 and is shown in the diagram below, where r = 4 and θ = 180°

Hence, –4 = 4∠180° in polar form

(c) –6 + j From the diagram below, r = 62 + 12 =37

1
and α tan −1  =
=  9.46° thus θ = 180° – 9.46° = 170.54°
6

Thus, –6 + j = 37∠170.54°

3. Express in polar form, leaving answers in surd form:

(a) – j3 (b) (– 2 + j)3 (c) j3(1 – j)


767 © 2014, John Bird
(a) –j3 From the diagram below, r = 3 and θ = –90°

Hence, – j3 = 3∠–90° in polar form

( −2 + j )
3
(b) = (–2 + j)(–2 + j)(–2 + j) = (4 – j2 – j2 + j 2 )(–2 + j)

= (3 – j4)(–2 + j) = –6 + j3 + j8 – j 2 4 = –2 + j11

 11 
From the diagram below, r = 22 + 112 =125 α tan −1 =
and =  79.70°
2
and θ = 180° – 79.70° = 100.30°

( −2 + j )
3
Hence, = –2 + j11 = 125∠100.30° in polar form

(c) j 3 (1 − j ) = (j)( j 2 )(1 – j) = –j(1 – j) = –j + j 2 = –1 – j

1
From the diagram below, r = 12 + 12 =2 and α= tan −1  =
 45°
1
and θ = 180° – 45° = 135°

Hence, j 3 (1 − j ) = –1 – j = 2∠ − 135°

4. Convert into (a + jb) form giving answers correct to 4 significant figures:

(a) 5∠30° (b) 3∠60° (c) 7∠45°

768 © 2014, John Bird


(a) 5∠30° = 5 cos 30° + j 5 sin 30° = 4.330 + j2.500

(b) 3∠60° = 3 cos 60° + j 3 sin 60° = 1.500 + j2.598

(c) 7∠45° = 7 cos 45° + j 7 sin 45° = 4.950 + j4.950

5. Convert into (a + jb) form giving answers correct to 4 significant figures:

(a) 6∠125° (b) 4∠π (c) 3.5∠–120°

(a) 6∠125° = 6 cos 125° + j 6 sin 125° = –3.441 + j4.915

(b) 4∠π = 4 cos π + j sin π = – 4.000 + j0 (Note that π is radians)

(c) 3.5∠–120° = 3.5 cos(–120°) + j 3.5 sin(–120°) = –1.750 – j3.031

6. Evaluate in polar form: (a) 3∠20° × 15∠45° (b) 2.4∠65° × 4.4∠–21°

(a) 3∠20°×15∠45° = 3 ×15∠(20° + 45°) = 45∠65°

(b) 2.4∠65°× 4.4∠ − 21° = 2.4 × 4.4∠(65° + −21°) = 10.56∠44°

7. Evaluate in polar form: (a) 6.4∠27° ÷ 2∠–15° (b) 5∠30° × 4∠80° ÷ 10∠–40°

6.4∠27° 6.4
(a) 6.4∠27° ÷ 2∠ − 15° = = ∠27° − −15° = 3.2∠42°
2∠ − 15° 2

5∠30°× 4∠80° 5 × 4
(b) 5∠30°× 4∠80° ÷ 10∠ − 40° = = ∠(30° + 80° − −40°) = 2∠150°
10∠ − 40° 10

π π
8. Evaluate in polar form: (a) 4 ∠ + 3∠ (b) 2∠120° + 5.2∠58° – 1.6∠– 40°
6 8

π π  π π  π π
(a) 4∠ + 3∠ =  4 cos + j 4sin  +  3cos + j 3sin  = (3.464 + j2) + (2.772 + j1.148)
6 8  6 6  8 8
= 6.236 + j3.148
From the diagram below, r = 6.2362 + 3.1482 =
6.986

769 © 2014, John Bird


 3.148 
and =θ tan −1  =  26.79° or 0.467 rad
 6.236 

π π
Hence, 4∠ + 3∠ = 6.986∠26.79° or 6.986∠0.467 rad
6 8
(b) 2∠120° + 5.2∠58° − 1.6∠ − 40°

= (2 cos 120° + j2 sin 120°) + (5.2 cos 58° + j5.2 sin 58°) – (1.6 cos(–40°) + j1.6 sin(–40°))

= (–1 + j1.732) + (2.756 + j4.410) – (1.226 – j1.028)

= –1 + j1.732 + 2.756 + j4.410 – 1.226 + j1.028

= 0.530 + j7.170

From the diagram below, r = 0.5302 + 7.1702 =


7.190

 7.170 
and =θ tan −1  =  85.77°
 0.530 

Hence, 2∠120° + 5.2∠58° − 1.6∠ − 40° = 7.190∠85.77°

770 © 2014, John Bird


EXERCISE 191 Page 519

1. Determine the resistance R and series inductance L (or capacitance C) for each of the following

impedances assuming the frequency to be 50 Hz:

(a) (3 + j8) Ω (b) (2 – j3) Ω (c) j14 Ω (d) 8∠– 60° Ω

(a) If Z = (3 + j8) Ω then resistance, R = 3 Ω and inductive reactance, X L = 8 Ω (since the j


term is positive)
8 8
X L = 2πfL = 8 hence, inductance, L = = = 0.0255 H or 25.5 mH
2π f 2π (50)
(b) If Z = (2 – j3) Ω then resistance, R = 2 Ω and capacitive reactance, X C = 3 Ω (since the j
term is negative)
1 1 1
XC = = 3 hence, capacitance, C = = = 1.061× 10−3 or 1061× 10−6
2π fC 2π f (3) 2π (50)(3)
= 1061 µF

(c) If Z = j14 Ω i.e. Z = (0 + j14) Ω then resistance, R = 0 Ω and X L = 14 Ω


14
i.e. 2πfL = 14 hence, inductance, L = = 0.04456 H or 44.56 mH
2π (50)
(d) If Z = 8∠ − 60°Ω = 8 cos(–60°) + j8 sin(–60°) = (4 – j6.928) Ω

Hence, resistance, R = 4 Ω and X C = 6.928 Ω


1 1
i.e. = 6.928 and capacitance, C = = 459.4 ×10−6 = 459.4 µF
2π fC 2π (50)(6.928)

2. Two impedances, Z1 = (3 + j6) Ω and Z 2 = (4 – j3) Ω are connected in series to a supply voltage of

120 V. Determine the magnitude of the current and its phase angle relative to the voltage

In a series circuit, total impedance, ZTOTAL= Z1 + Z 2 = (3 + j6) + (4 – j3) = (7 + j3) Ω

3
= 7 2 + 32 ∠ tan −1  
7
= 7.616∠23.20° Ω
V 120∠0°
Since voltage V = 120∠0° V, then current, I = = = 15.76∠– 23.20° A
Z 7.616∠23.20°
i.e. the current is 15.76 A and is lagging the voltage by 23.20°

771 © 2014, John Bird


3. If the two impedances in Problem 2 are connected in parallel, determine the current flowing and its

phase relative to the 120 V supply voltage.

In a parallel circuit shown below, the total impedance ZT is given by:


1 1 1 1 1 3 − j 6 4 + j3 3 6 4 3
= + = + = + = −j + +j
ZT Z1 Z 2 3 + j 6 4 − j 3 32 + 62 42 + 32 45 45 25 25
1
i.e. = admittance, YT = 0.22667 – j0.01333 = 0.2271∠–3.37° siemen
ZT

V
Current, I = = VY
=T (120∠0°)(0.2271∠ − 3.37=
°) 27.25∠ − 3.37° A
ZT
i.e. the current is 27.25 A and is lagging the voltage by 3.37°

4. A series circuit consists of a 12 Ω resistor, a coil of inductance 0.10 H and a capacitance of 160 µF.

Calculate the current flowing and its phase relative to the supply voltage of 240 V, 50 Hz. Determine

also the power factor of the circuit.

R = 12 Ω , inductive reactance, X L = 2πfL = 2π(50)(0.10) = 31.416 Ω

1 1
=
and capacitive reactance, XC = = 19.894 Ω
2π f C 2π ( 50 )(160 ×10−6 )

Hence, impedance, Z = R + j( X L − X C ) = 12 + j(31.416 – 19.894) = (12 + j11.52) Ω = 16.64∠43.83° Ω

V 240∠0°
Current flowing, I = = = 14.42∠– 43.83° A
Z 16.64∠43.83°

Phase angle = 43.83° lagging (i.e. I lags V by 43.83°)

Power factor = cos ϕ = cos 43.83° = 0.721

772 © 2014, John Bird


5. For the circuit shown, determine the current I flowing and its phase relative to the applied voltage.

1 1 1 1 1 1 1 30 + j 20 40 − j 50 1
= + + = + + = + +
ZT Z1 Z 2 Z 3 30 − j 20 40 + j 50 25 302 + 202 402 + 502 25
30 20 40 50 1
= +j + −j +
1300 1300 4100 4100 25
1
i.e. = admittance,YT = 0.07283 + j0.00319 = 0.0729∠2.51° S
ZT
V
Current, I = = VYT = (200∠0°)(0.0729∠2.51°) = 14.6∠2.51° A
ZT
i.e. the current is 14.6 A and is leading the voltage by 2.51°

6. Determine, using complex numbers, the magnitude and direction of the resultant of the coplanar forces

given below, which are acting at a point. Force A, 5 N acting horizontally, Force B, 9 N acting at an

angle of 135° to force A, Force C, 12 N acting at an angle of 240° to Force A.

Resultant force = FA + FB + FC = 5∠0° + 9∠135° + 12∠240°

= (5 + j0) + (–6.364 + j6.364) + (–6 – j10.392)

= 5 + j0 – 6.364 + j6.364 – 6 – j10.392)

= –7.364 – j4.028

= 8.394∠– 151.32° or 8.394∠208.68° N

Hence, the magnitude of the force that has the same effect as the three forces acting separately is:

8.392 N and its direction is 208.68° to the horizontal (i.e. from Force A)

773 © 2014, John Bird


7. A delta-connected impedance Z A is given by:

Z1Z 2 + Z 2 Z 3 + Z 3 Z1
ZA =
Z2

Determine Z A in both Cartesian and polar form given Z 1 = (10 + j0) Ω, Z2 = (0 – j10) Ω and

Z 3 = (10 + j10) Ω

Z1Z 2 + Z 2 Z 3 + Z 3 Z1 (10 + j 0)(0 − j10) + (0 − j10)(10 + j10) + (10 + j10)(10 + j 0)


ZA =
Z2 (0 − j10)
− j100 − j100 − j 2100 + 100 + j100 200 − j100 200 j100
= = = −
− j10 − j10 − j10 − j10
j 200
= + 10 = (10 + j 20) Ω
10
 20 
From the diagram below, r = 102 + 202 = 22.36 and =θ tan −1  =  63.43°
 10 

Hence, Z=
A (10 + j 20) Ω
= 22.36∠63.43°Ω

8. In the hydrogen atom, the angular momentum p of the de Broglie wave is given by

 jh 
pψ = –   (± jmψ). Determine an expression for p.
 2π 

 jh  j h  ± j m Ψ   jh h 2
If pΨ = −   ( ± j m Ψ ) then p = −   =−  (± j m) =
− ( j )( ± m )
 2π   2π  Ψ   2π  2π
h mh
= ( ±m ) = ±
2π 2π

774 © 2014, John Bird


9. An aircraft P flying at a constant height has a velocity of (400 + j300)km/h. Another aircraft Q at the

same height has a velocity of (200 – j600) km/h. Determine (a) the velocity of P relative to Q, and

(b) the velocity of Q relative to P. Express the answers in polar form, correct to the nearest km/h.

(a) The velocity of P relative to Q = vP − vQ = (400 + j300) – (200 – j600)

= 400 + j300 – 200 + j600

= 200 + j900 = 922∠77.47°

i.e. the velocity of P relative to Q is 922 km/h at 77.47°

(b) The velocity of Q relative to P = vQ − vP = (200 – j600) – (400 + j300)

= 200 – j600 – 400 – j300

= –200 – j900 = 922∠–102.53°

i.e. the velocity of Q relative to P is 922 km/h at –102.53°

10. Three vectors are represented by P, 2∠30°, Q, 3∠90° and R, 4∠–60°. Determine in polar form the

vectors represented by (a) P + Q + R, (b) P – Q – R.

(a) P + Q + R = 2∠30° + 3∠90° + 4∠–60° = (1.732 + j1) + (0 + j3) + (2 – j3.464)

= (3.732 + j0.536) = 3.770∠8.17°

(b) P – Q – R = 2∠30° – 3∠90° – 4∠–60° = (1.732 + j1) – (0 + j3) – (2 – j3.464)

= (–0.268 + j1.464)

 1.464 
α tan −1  =
From the diagram below, r = 1.488 and =  79.63°
 0.268 

and θ 180° − 79.63=


= ° 100.37°

775 © 2014, John Bird


Hence, P – Q – R = 1.488∠100.37°

( R3 )( − jX C )
11. In a Schering bridge circuit, =
Zx ( RX − jX C ) , Z 2 = − jX C2 , Z 3 =
3
and
( R3 − jX C )
X
3

1
Z 4 = R4 where X C = . At balance: ( Z X )( Z 3 ) = ( Z 2 )( Z 4 ) .
2π fC
C3 R4 C2 R3
Show that at balance RX = and C X =
C2 R4

Since ( Z X )( Z3 ) = ( Z 2 )( Z 4 )
 ( R3 )( − jX C3 ) 
then ( RX − jX C X )  =( − jX C2 )( R4 )
 R3 − jX C3 

Thus,
( R3 − jX C3 )( − jX C2 )( R4 )
( RX − jX C X ) =
( R3 )( − jX C3 )
− j R3 X C2 R4 j 2 X C3 X C2 R4
i.e. ( RX − jX
= CX ) +
( R3 )( − jX C3 ) ( R3 )( − jX C3 )
X C2 R4 X C2 R4
i.e. ( RX − jX C X ) = −
X C3 ( R3 ) (− j )
X C2 R4 X C2 R4
= +
X C3 j R3

X C2 R4 X C R4
i.e. ( RX − jX C X ) = −j 2
X C3 R3
1
R4
X C2 R4 2π fC2 2π fC3
Equating the real parts gives: =
RX = = R4
X C3 1 2π fC2
2π fC3
C3 R4
i.e. RX =
C2
X C R4
Equating the imaginary parts gives: − X C X =
− 2
R3
1
R4
1 2π fC2 R4
i.e. = =
2π fC X R3 2π fC2 R3
C2 R3
from which, CX =
R4

776 © 2014, John Bird


500
12. An amplifier has a transfer function T given by T = where ω is the angular
1 + jω ( 5 ×10−4 )

frequency. The gain of the amplifier is given by the modulus of T and the phase is given by the

argument of T. If ω = 2000 rad/s, determine the gain and the phase (in degrees).

500 500 500


When ω = 2000 rad/s , transfer function T=
= =
1 + jω ( 5 ×10 ) 1 + j (2000)(5 ×10 ) 1 + j1
−4 −4

500 (500)(1 − j1) 500 − j 500 500 − j 500 500 500


Hence, =
T= = = = −j
1 + j1 (1 + j1)(1 − j1) 12 + 12 2 2 2

= 250 – j250 = 353.6∠– 45°

Hence, the gain of the amplifier = 353.6 and the phase is –45°

VS
13. The sending end current of a transmission line is given by I S = tanh PL . Calculate the
Z0
value of the sending current, in polar form, given VS = 200 V , Z 0 =560 + j 420 Ω , P = 0.20
and L = 10

VS 200 200 tanh 2


=
Sending current, IS =
tanh PL tanh ( 0.20=
×10 )
Z0 (560 + j 420) (560 + j 420)

192.8 (192.8)(560 − j 420) (192.8)(560 − j 420)


=
= =
(560 + j 420) (560 + j 420)(560 − j 420) 5602 + 4202

(192.8) ( 700∠ − 36.87° )


= = 0.275∠ − 36.87° A
490000

i.e. the sending end current, I S = 275∠ − 36.87° mA

777 © 2014, John Bird

Das könnte Ihnen auch gefallen